Download as pdf or txt
Download as pdf or txt
You are on page 1of 2

KEITH DEVLIN: Introduction to Mathematical Thinking (Winter 2014) Problem Set Week 8

This problem set focuses on material covered in Week 8 (Lecture 10), so I recommend you to watch
the lecture and attempt Assignment 10 (both parts) before submitting your answers. The deadline for
completing (and submitting) the problem set is Monday March 31 at 9:00AM US-PST. Note that you
can save your entries as you work through the problems, and can change them at any time prior to
submission, but once you submit your answers no further changes are possible.
1. Say which of the following are true. (Leave the box empty to indicate that its false.)
A set A of reals can have at most one least upper bound.
If a set A of reals has a lower bound, it has innitely many lower bounds.
If a set A of reals has both a lower bound and an upper bound, then it is nite.
0 is the least upper bound of the set of negative integers, considered as a subset of the reals.
2. Which of the following say that b is the greatest lower bound of a set A of reals? (Leave the box
empty to indicate that it does not say that.)
b a for all a A and if c a for all a A, then b c.
b a for all a A and if c a for all a A, then b > c.
b < a for all a A and if c < a for all a A, then b c.
b < a for all a A and if c a for all a A, then b c.
b a for all a A and if > 0 there is an a A such that a < b +.
3. The Sandwich Theorem (also sometimes called the Squeeze Theorem) says that if {a
n
}

n=1
, {b
n
}

n=1
,
{c
n
}

n=1
are sequences such that, from some point n
0
onwards,
a
n
b
n
c
n
,
and if
lim
n
a
n
= L , lim
n
c
n
= L,
then {b
n
}

n=1
is convergent and
lim
n
b
n
= L.
Taking the Sandwich Theorem to be correct (which it is), grade the following proof using the course
rubric.
Theorem lim
n
sin
2
n
3
n
= 0
Proof: For any n,
0
sin
2
n
3
n

1
3
n
Clearly, lim
n
1
3
n
= 0. Hence, by the Sandwich Theorem,
lim
n
sin
2
n
3
n
= 0
as required.
1
4. Is the following proof of the Sandwich Theorem correct? Grade it according to the course rubric.
Theorem (Sandwich Theorem) Suppose {a
n
}

n=1
, {b
n
}

n=1
, {c
n
}

n=1
are sequences such that, from
some point n
0
onwards,
a
n
b
n
c
n
.
Suppose further that
lim
n
a
n
= L , lim
n
c
n
= L.
Then {b
n
}

n=1
is convergent and
lim
n
b
n
= L.
Proof: Since lim
n
a
n
= L, we can nd an integer n
1
such that
n n
1
|a
n
L| <
Since lim
n
c
n
= L, we can nd an integer n
2
such that
n n
2
|c
n
L| <
Let M = max{n
0
, n
1
, n
2
}. Then
n M ( < a
n
L < ) ( < c
n
L < )
< a
n
L b
n
L c
n
L < (using a
n
b
n
c
n
)
< b
n
L <
|b
n
L| <
By the denition of a limit, this proves that {b
n
}

n=1
is convergent and lim
n
b
n
= L, as required.
5. Evaluate this purported proof, and grade it according to the course rubric.
Theorem lim
n
n + 1
2n + 1
=
1
2
.
Proof: Let > 0 be given. Choose N large enough so that N
1
2
.
Then, for n N,

n + 1
2n + 1

1
2

2(n + 1) (2n + 1)
2(2n + 1)

1
2(2n + 1)

=
1
2(2n + 1)
<
1
2n + 1
<
1
2n

1
2N

By the denition of a limit, this proves the theorem.
2

You might also like